1answer.
Ask question
Login Signup
Ask question
All categories
  • English
  • Mathematics
  • Social Studies
  • Business
  • History
  • Health
  • Geography
  • Biology
  • Physics
  • Chemistry
  • Computers and Technology
  • Arts
  • World Languages
  • Spanish
  • French
  • German
  • Advanced Placement (AP)
  • SAT
  • Medicine
  • Law
  • Engineering
gayaneshka [121]
3 years ago
15

A manufacturer earns Rs4500 in the first month and Rs 6000 in second month. On plotting these points a manufacturer observes a l

inear function. Find the linear function that fits in data?
Mathematics
1 answer:
UkoKoshka [18]3 years ago
7 0

Answer:

f(x) = 1500x +3000

Step-by-step explanation:

Given

Represent the months with x and the earnings with y.

So:

(x_1,y_1) = (1,4500)

(x_2,y_2) = (2,6000)

Required

Determine the linear function

First, calculate slope (m)

m = \frac{y_2 - y_1}{x_2 - x_1}

m = \frac{6000 - 4500}{2 - 1}

m = \frac{1500}{1}

m = 1500

The equation is then calculated using:

y = m(x - x_1) + y_1

y = 1500(x - 1) + 4500

y = 1500x - 1500 + 4500

y = 1500x +3000

As a function, we have:

f(x) = 1500x +3000

You might be interested in
Neil drives at an average speed of 60 miles/hour to reach
nikklg [1K]

Answer:

 480/(x+60) ≤ 7

Step-by-step explanation:

We can use the relations ...

 time = distance/speed

 distance = speed×time

 speed = distance/time

to write the required inequality any of several ways.

Since the problem is posed in terms of time (7 hours) and an increase in speed (x), we can write the time inequality as ...

 480/(60+x) ≤ 7

Multiplying this by the denominator gives us a distance inequality:

 7(60+x) ≥ 480 . . . . . . at his desired speed, Neil will go no less than 480 miles in 7 hours

Or, we can write an inequality for the increase in speed directly:

 480/7 -60 ≤ x . . . . . . x is at least the difference between the speed of 480 miles in 7 hours and the speed of 60 miles per hour

___

Any of the above inequalities will give the desired value of x.

8 0
3 years ago
Marya plans to participate in a 20 km walk-a-thon. To prepare, she walked 4 km on Monday. She plans to increase her walk by 2 km
Afina-wow [57]

Answer: 8 weeks

Step-by-step explanation:

20 - 4 = 16

16 / 2 = 8

7 0
3 years ago
Read 2 more answers
Hello, Brainly community!
ioda

Answer:

(B)  \displaystyle \frac{W(3.1) - W(2.9)}{0.2}

General Formulas and Concepts:

<u>Calculus</u>

Limits

Derivatives

  • The definition of a derivative is the slope of the tangent line.

Derivative Notation

Instantaneous Rates

  • Tangent Line: \displaystyle f'(x) = \frac{f(b) - f(a)}{b - a}

Step-by-step explanation:

Since we are trying to find a <em>rate</em> at which W(t) changes, we must find the <em>derivative</em> at <em>t</em> = 3.

We are given 2 close answer choices that would have the same <em>numerical</em> answer but different <em>meanings</em>:

  1. (A)  \displaystyle  \lim_{t \to 3} W(t)
  2. (B)  \displaystyle \frac{W(3.1) - W(2.9)}{0.2}

If we look at answer choice (A), we see that our units would simply just be volume. It would not have the units of a rate of change. Yes, it may be the closest numerically correct answer, but it does not tell us the <em>rate</em> at which the volume would be changing and it is not a derivative.

If we look at answer choice (B), we see that our units would be cm³/s, and that is most certainly a rate of change. Answer choice (B) is also a <em>derivative</em> at <em>t</em> = 3, and a derivative tells us what <em>rate</em> something is changing.

∴ Answer choice (B) will give us the best estimate for the value of the instantaneous rate of change of W(t) when <em>t</em> = 3.

Topic: AP Calculus AB/BC (Calculus I/I + II)

Unit: Differentiation

Book: College Calculus 10e

8 0
3 years ago
Subtract 4 1/3 -8 enter your answer as a simplified mixed number by filling in the boxes
harina [27]
-3 2/3 I just changed the mixed number into a improper fraction
6 0
2 years ago
Read 2 more answers
Michaels account overdrafts $75.00. write an absolute value to represent the overdraft
Klio2033 [76]

Overdrafting is when you deposit you're money in a bank account so it could either be +75 or -75 both of them might be correct.

8 0
3 years ago
Other questions:
  • The table shows the cost of renting movies online.
    13·2 answers
  • 1. From a circular sheet of radius 18 cm, two circles of radii 4.5 cm and a rectangle of length 4 cm and breadth 1 cm are remove
    12·1 answer
  • PLEASE HELP FAST!! WOULD BE VERY APPRECIATED!
    14·1 answer
  • What is the area of a regular hexagon with a side length of 5in and an apothem of 4.33in?
    10·1 answer
  • A company produces refrigerators for beer kegs. The refrigerators are supposed to maintain a mean temperature of 43°F, ideal for
    6·1 answer
  • When will the pain end
    6·2 answers
  • Please help me in this! you get 30 points!<br>​
    15·2 answers
  • The volume of a pyramid is 47m3. The height of the pyramid is 6 meters. What is the area of its base?
    13·2 answers
  • Y’all please help on these 2 questions !! asap
    8·1 answer
  • Micah wants to buy a new guitar with his debit card. Which statement is probably true?
    10·1 answer
Add answer
Login
Not registered? Fast signup
Signup
Login Signup
Ask question!